Sabatine. Casos clínicos_1ed

Capítulo 1 Cardiología 9

¿Cuál de los siguientes es el mejor paso diagnóstico a seguir? A. Medición de presiones intracardíacas mediante catéter en la arteria pulmonar (AP) B. TC sin contraste de abdomen/pelvis C. Repetición urgente de angiografía coronaria para evaluar permeabilidad de endoprótesis D. Ecocardiografía transtorácica urgente 21. Hombre de 65 años de edad acude al centro de intervencionismo coronario por dolor torácico sub- esternal opresivo que comenzó mientras quitaba nieve. En el ECG se observan elevaciones del seg- mento ST de 2 mm en las derivaciones anteriores. Inicia la terapia apropiada, pero requiere apoyo vasopresor por evolución de choque cardiogénico. Dadas las inclemencias del tiempo, el hospital más cercano con capacidad de intervención coronaria percutánea está a más de 3 h de distancia. ¿Cuál de las siguientes serían las estrategias de gestión más adecuadas? A. Administrar alteplasa dentro de los 30 min posteriores a la llegada al hospital y proceder con el traslado al centro con capacidad de intervención coronaria percutánea si hay evidencia de reperfusión fallida o reoclusión B. Administrar alteplasa dentro de los 30 min posteriores a la llegada al hospital y proceder con el traslado urgente a un centro con capacidad para intervención coronaria percutánea C. Administrar alteplasa dentro de los 30 min posteriores a la llegada al hospital y trasladar al centro con capacidad para intervención coronaria percutánea dentro de las 3-24 h D. Iniciar el traslado inmediato al centro con capacidad de intervención coronaria percutánea 22. Hombre de 6 1 años de edad acude con dolor torácico subesternal implacable que inició hace 30 min, con elevación del segmento ST inferior en el ECG sin ninguna otra modi cación. Hemodinámicamente estable. En la angiografía coronaria urgente se detecta oclusión en la arteria coronaria derecha media. También se observa lesión descendente anterior izquierda proximal del 80%. Además de la endoprótesis vascular en la arteria coronaria derecha, ¿cuál sería el mejor tratamiento? A. Programar la colocación de una endoprótesis vascular en la arteria descendente anterior izquierda en los próximos 45 días B. No hacer planes de revascularización C. Realizar reserva de ¢ujo fraccional en la lesión descendente anterior izquierda D. Programar prueba de esfuerzo ambulatoria para evaluar arteria descendente anterior izquierda 23. Mujer de 78 años de edad con hipertensión presentó un IM sin elevación del ST y se le colocó una endoprótesis vascular liberadora de fármacos en la arteria coronaria derecha. La concentración de lipoproteínas de baja densidad (LDL, low density lipoproteins ), de 1 05 mg/dL, es tratada con estatinas de alta intensidad. ¿Qué cambios, en caso de ser necesarios, se deben realizar en su régimen de reducción de lípidos antes del alta? A. Agregar un medicamento secuestrante de ácidos biliares SAMPLE B. Agregar un brato C. Agregar ezetimiba D. Continuar solo con estatinas de alta intensidad 24. Mujer de 83 años de edad con antecedentes de hiperlipidemia presentó un IM sin elevación del ST y en la arteriografía coronaria hay disminución del ¢ujo. Se utilizó manejo conservador y se inició una estatina de intensidad moderada a alta y un bloqueador β . Inicialmente, ¿cuál de los siguientes medicamentos debería incluir su régimen antiplaquetario de forma ideal? A. Ácido acetilsalicílico

CARDIOLOGÍA

B. Prasugrel C. Ticagrelor

D. A y B E. A y C

Made with FlippingBook - professional solution for displaying marketing and sales documents online